Naked Science Forum

On the Lighter Side => New Theories => Topic started by: mxplxxx on 17/12/2018 05:35:32

Title: How is the quantum double slit experiment set up?
Post by: mxplxxx on 17/12/2018 05:35:32
In the famous double slit experiment of quantum physics a photon fired at a wall with two slits in it appears to behave as a wave. Is this at all dependent on where the photon is fired from? For example, If the photon is fired at the part of the wall between the two slits, is it absorbed by the wall? If it is always fired at the middle of a slit, does it always pass through that slit? What is the wavefunction of the photon in the experiment? What does the wave in the experiment look like?
Title: Re: How is the quantum double slit experiment set up?
Post by: RobC on 17/12/2018 11:56:06
Five questions there. I have never seen a plausible answer to most of them.
Title: Re: How is the quantum double slit experiment set up?
Post by: Bored chemist on 17/12/2018 17:32:07
Because of diffraction effects the beam is never "narrow" compared to the slits' separation.

You can , if you want, set the particles moving from very close to one of the slits or to the wall between them.
The maths is more complicated.
https://en.wikipedia.org/wiki/Near_and_far_field#Near-field_diffraction
I have never seen a plausible answer to most of them.
Did you look in the right places?
Title: Re: How is the quantum double slit experiment set up?
Post by: mxplxxx on 17/12/2018 21:08:51
Because of diffraction effects the beam is never "narrow" compared to the slits' separation.

You can , if you want, set the particles moving from very close to one of the slits or to the wall between them.
The maths is more complicated.
https://en.wikipedia.org/wiki/Near_and_far_field#Near-field_diffraction
I have never seen a plausible answer to most of them.
Did you look in the right places?
I agree with Robc, I have never seen a satisfactory explanation of this experiment. And I have looked high and low. Given you have not provided any references, it would seem you are in the same boat. I am currently reading a book "Farewell To Reality" by Jim Baggott that criticizes much of modern Physics theory. Incredibly he starts the book by talking about the double slit experiment as if it were a proven basis for quantum physics. We need a modern version of the experiment that answers my questions. The fact that we don't have one may indicate that Quantum physics has been founded on an urban myth.
Title: Re: How is the quantum double slit experiment set up?
Post by: Bored chemist on 17/12/2018 21:30:31
Given you have not provided any references,
This
https://en.wikipedia.org/wiki/Near_and_far_field#Near-field_diffraction
is a reference.

It may be that you
looked high and low.
but you didn't find anything because you didn't recognise it when it was staring you in the face.
Incredibly he starts the book by talking about the double slit experiment as if it were a proven basis for quantum physics.
That's because it is.
It's a simple experiment that demonstrates the nature of light.
We need a modern version of the experiment that answers my questions.
If you really think so, then do the experiment (for the zillionth time)
Thomas Young managed to do it in 1801
It's a high-school physics experiment today.
What is stopping you?
Are you scared it might work?
Title: Re: How is the quantum double slit experiment set up?
Post by: mxplxxx on 17/12/2018 21:46:08
I am after answers to my questions and this is not happening, Beats me why! I am talking about a single photon, not electromagnetic waves which are not quantum entities.
Title: Re: How is the quantum double slit experiment set up?
Post by: PmbPhy on 17/12/2018 22:11:00
In the famous double slit experiment of quantum physics a photon fired at a wall with two slits in it appears to behave as a wave. Is this at all dependent on where the photon is fired from? For example, If the photon is fired at the part of the wall between the two slits, is it absorbed by the wall? If it is always fired at the middle of a slit, does it always pass through that slit? What is the wavefunction of the photon in the experiment? What does the wave in the experiment look like?
One can use a laser beam aimed at a double sit, which can be made by using an opaque tape in a sheet of glass. Aim the been at the center of the slits. Make sure that the slits are close enough, i.e. less than half a wave length. If a photon hits the slit wall itself then its not considered with the data collected on the other side of the slit. This won't happen if the slits are close enough to each other.

A wave function of a photon is an exponential function. I forget the form it has.
Title: Re: How is the quantum double slit experiment set up?
Post by: mxplxxx on 17/12/2018 22:27:27
In the famous double slit experiment of quantum physics a photon fired at a wall with two slits in it appears to behave as a wave. Is this at all dependent on where the photon is fired from? For example, If the photon is fired at the part of the wall between the two slits, is it absorbed by the wall? If it is always fired at the middle of a slit, does it always pass through that slit? What is the wavefunction of the photon in the experiment? What does the wave in the experiment look like?
One can use a laser beam aimed at a double sit, which can be made by using an opaque tape in a sheet of glass. Aim the been at the center of the slits. Make sure that the slits are close enough, i.e. less than half a wave length. If a photon hits the slit wall itself then its not considered with the data collected on the other side of the slit. This won't happen if the slits are close enough to each other.

A wave function of a photon is an exponential function. I forget the form it has.
Thx. Nowhere can I find the wave function of a photon. Strange.
Title: Re: How is the quantum double slit experiment set up?
Post by: Bored chemist on 17/12/2018 23:13:13
Nowhere can I find the wave function of a photon. Strange.
Eqn 58 here
http://iopscience.iop.org/article/10.1088/1367-2630/9/11/414

If it turns out that you don't understand it, that just tells us there was no point in you looking for it.
Title: Re: How is the quantum double slit experiment set up?
Post by: Bored chemist on 17/12/2018 23:15:08
I am after answers to my questions and this is not happening,
And, when you get answers you simply ignore them,
Title: Re: How is the quantum double slit experiment set up?
Post by: PmbPhy on 18/12/2018 01:04:10
Thx. Nowhere can I find the wave function of a photon. Strange.
Not really. A photon is a relativistic particle and not fully treated in quantum mechanics but in quantum electrodynamics. See QED by Richard Feynman and you can read all about it. The state vector is found in Quantum Mechanics II: A Second Course on Quantum Theory, page 341.
Title: Re: How is the quantum double slit experiment set up?
Post by: PmbPhy on 18/12/2018 01:06:31
Quote from: Bored chemist
If it turns out that you don't understand it, that just tells us there was no point in you looking for it.
Total nonsense. If a member doesn't understand it then they should look for it to learn about it.
Title: Re: How is the quantum double slit experiment set up?
Post by: mxplxxx on 18/12/2018 01:12:53
Nowhere can I find the wave function of a photon. Strange.
Eqn 58 here
http://iopscience.iop.org/article/10.1088/1367-2630/9/11/414

If it turns out that you don't understand it, that just tells us there was no point in you looking for it.
I have no intention of trying to follow complex maths (although I could, I have maths at university level standard), I just want an equation, but it seem that such a beast does not exist and if that is the case how can it be hypothesized that a photon exists as a wave in the double slit experiment?
Title: Re: How is the quantum double slit experiment set up?
Post by: mxplxxx on 18/12/2018 01:20:33
One can use a laser beam aimed at a double sit, which can be made by using an opaque tape in a sheet of glass.
This would not result in a single photon at a time would it?
Title: Re: How is the quantum double slit experiment set up?
Post by: PmbPhy on 18/12/2018 01:47:35
One can use a laser beam aimed at a double sit, which can be made by using an opaque tape in a sheet of glass.
This would not result in a single photon at a time would it?
It depends on whether you put a filter right out of laser to filter (polarizer) photons out. Theoretically you could reduce the rate to one at a time.
Title: Re: How is the quantum double slit experiment set up?
Post by: mxplxxx on 18/12/2018 01:49:43

* Capture.PNG (522.74 kB . 2950x1840 - viewed 5822 times)Here is a good video of the experiment.https://www.youtube.com/watch?v=GzbKb59my3U (https://www.youtube.com/watch?v=GzbKb59my3U)
also https://www.youtube.com/watch?v=FzVT56ZPLEo (https://www.youtube.com/watch?v=FzVT56ZPLEo)

The problem with maths is that it has difficulty describing complex systems. These days I look for computer simulations, particularly using using HSMs (hierarchical state machines), to understand physics. Because I am a software developer, I can examine the code of such systems to gain a deeper understanding of them. A HSM of a hydrogen atom is illustrated below:

Title: Re: How is the quantum double slit experiment set up?
Post by: Bored chemist on 18/12/2018 10:28:36
I just want an equation, but it seem that such a beast does not exist
What is your problem?
There is an equation.
I cited it for you.
And now you pretend it doesn't exist.
I have no intention of trying to follow complex maths
So, you want stuff, but are not prepared to do any work.
How well would that work out anywhere else in life?



This would not result in a single photon at a time would it?
It does if you move the laser far enough away from the slits.

The experiment has been done. you are welcome to do it again, but what's the point? The outcome won't change.

Title: Re: How is the quantum double slit experiment set up?
Post by: mxplxxx on 18/12/2018 12:19:03
BoredChemist, life would likely be much kinder to you i f you concentrated on the questions, not the personalities. We are here to help each other, not criticize. You may be less bored (depressed?) if you make this transition.

LIke Einstein, I don't need maths to understand the universe. The hideous complexity of maths in modern physics comes about because the concepts are difficult to translate into maths. The concepts came before the maths and this is what I am interested in.
Title: Re: How is the quantum double slit experiment set up?
Post by: Bored chemist on 18/12/2018 12:46:46
BoredChemist, life would likely be much kinder to you i f you concentrated on the questions
I'm hardly going to take advice on where to focus my attentions from someone who doesn't recognise that the question was already answered.

Now, given that I already told you what the wave function is, why are you still saying
I just want an equation, but it seem that such a beast does not exist
The concepts came before the maths and this is what I am interested in.
OK, so, you want an equation; but not maths.

Do you see how that might be a problem?
Title: Re: How is the quantum double slit experiment set up?
Post by: mxplxxx on 18/12/2018 13:00:29
OK, so, you want an equation; but not maths.

Do you see how that might be a problem?

No, tell me why it may be a problem and define problem in this instance.

And tell me again pls, what is the wave function for a proton.
Title: Re: How is the quantum double slit experiment set up?
Post by: Bored chemist on 18/12/2018 13:15:44
And tell me again pls, what is the wave function for a proton.
Why don't you just read the thread again?
I guess it's Christmas so, here it is.
Eqn 58 here
http://iopscience.iop.org/article/10.1088/1367-2630/9/11/414


No, tell me why it may be a problem and define problem in this instance.
Because equations (in this use of the word) are part of maths.
So you can't have an equation, but not maths.

This definition will probably do.
problem
/ˈprɒbləm/
noun
1.
a matter or situation regarded as unwelcome or harmful and needing to be dealt with and overcome.
"they have financial problems"
Title: Re: How is the quantum double slit experiment set up?
Post by: mxplxxx on 18/12/2018 13:30:39
Why don't you just read the thread again?
I guess it's Christmas so, here it is

A wave function is an equation.I just want to know what this equation is for a photon. If, as I suspect, it is not known then how can the conclusion be arrived that the results of the double slit experiment are due to a photon existing as both a wave and a particle.
Title: Re: How is the quantum double slit experiment set up?
Post by: Bored chemist on 18/12/2018 17:08:44
If, as I suspect, it is not known
Which part of this reference
Eqn 58 here
http://iopscience.iop.org/article/10.1088/1367-2630/9/11/414

do you not understand.
The 58th (numbered) equation in that paper is the wavefunction of a photon.

How have you come to the conclusion that it is unknown- are you just trolling?
Title: Re: How is the quantum double slit experiment set up?
Post by: Bored chemist on 18/12/2018 17:13:26
how can the conclusion be arrived that the results of the double slit experiment are due to a photon existing as both a wave and a particle.
From logic.
In the video you can see that an interference pattern is created- that clearly requires wave behaviour.
You can also see that the same pattern is generated when you count photons (as a function of position).
To get single photons (where only 1 is present in the apparatus at a time) and an interference pattern, requires the light to have both the properties of a wave and of  particle.

The wave function's not going to make that any clearer; especially not to someone who says


LIke Einstein, I don't need maths to understand the universe.
Title: Re: How is the quantum double slit experiment set up?
Post by: mxplxxx on 18/12/2018 20:49:31
Thank you Bored Chemist for clarifying Eq 58. It is pretty much incomprehensible to me. What are the concepts it embodies? What does it look like? How does it explain the results of the double slit experiment?

Despite this, the quantum wave function of a photon seems to remain a controversial proposition in physics.  https://arxiv.org/abs/quant-ph/0508202 (https://arxiv.org/abs/quant-ph/0508202) 

The comparison with Einstein is unfortunate, to say the least :). Apparently he had difficulty with the more complex maths involved in relativity. https://www.quora.com/What-type-of-Math-did-Einstein-use (https://www.quora.com/What-type-of-Math-did-Einstein-use). Ditto me. I have university maths but struggle with stuff like eq 58. 
Title: Re: How is the quantum double slit experiment set up?
Post by: Bored chemist on 18/12/2018 22:07:32
The comparison with Einstein is unfortunate,
Well, next time, don't make it.
How does it explain the results of the double slit experiment?
Who said it did?

However the double slit experiment really does show  quantum duality.
It is pretty much incomprehensible to me.
I addressed that earlier.
If it turns out that you don't understand it, that just tells us there was no point in you looking for it.
Title: Re: How is the quantum double slit experiment set up?
Post by: yor_on on 18/12/2018 23:01:57
Maybe this one?

I know what you mean mxplxxx . You want to make it a part of you. But I would suggest that you start with the people that are here, then look around. We all get our own intuitions of how SpaceTime works, and mathematics do help. But if you're not a mathematician it should still be explainable. Another thing is trust. If you tell people that made it their field of science that they don't 'know' because they haven't found a way to make it work for you then you should stop trusting anyone :) Not good if I may so, the world runs on trust.

Another point is that science actually try to do away with trust, building a world from logic. But then you will need the mathematics to check if you agree.



Title: Re: How is the quantum double slit experiment set up?
Post by: mxplxxx on 19/12/2018 01:45:01
Maybe this one?

I know what you mean mxplxxx . You want to make it a part of you. But I would suggest that you start with the people that are here, then look around. We all get our own intuitions of how SpaceTime works, and mathematics do help. But if you're not a mathematician it should still be explainable. Another thing is trust. If you tell people that made it their field of science that they don't 'know' because they haven't found a way to make it work for you then you should stop trusting anyone :) Not good if I may so, the world runs on trust.

Another point is that science actually try to do away with trust, building a world from logic. But then you will need the mathematics to check if you agree.





Not sure if replying to your post is going to achieve anything but here goes. I am trying to understand reality. People seem to have an innate need to do this. I also try and use concepts from Physics to build better software. I have discovered that simulating a photon in software seems impossible because of the limitations of current physics theory. To get to this conclusion, I have had to do an enormous amount of research. It seems it is something the physics community don't want the rest of the world to know. I can understand this. People are very protective of their livelihoods/beliefs but trust suffers as a result.



Title: Re: How is the quantum double slit experiment set up?
Post by: yor_on on 19/12/2018 08:47:05
No, nobody is hiding anything. But nobody has a final answer, what is used is the 'standard theory', and it's not final either. Lights duality is one of the thing physics try to make sense of using concepts as wave packets. And it makes sense mathematically but the idea of a 'photon' being one irritates me too, if that's what you're thinking of? And no again, each field of mathematics has its own way to describe reality. Einstein didn't have problem as such with mathematics, he had problems with finding the correct description, and learning the mathematics used for it as I remember. If you look at field theory and the mathematics behind it's not the one used for strings, and it will take you decades to understand just one branch. Some people give a lifetime to solve a mathematical problem. It's not as easy as you think, it's work.
Title: Re: How is the quantum double slit experiment set up?
Post by: yor_on on 19/12/2018 08:53:17
Maybe you should tell us where your problem is when simulating a 'photon' in software? What is it the software get stuck on?
Title: Re: How is the quantum double slit experiment set up?
Post by: mxplxxx on 19/12/2018 10:13:53
Maybe you should tell us where your problem is when simulating a 'photon' in software? What is it the software get stuck on?
This would not be possible without an understanding of my Hierarchical Abstraction Framework (HAF) software that I am hoping will be able to simulate the universe. In this software, a photon is a particular type of event (all bosons are events) that changes the state of objects (fermions) that it interacts with in the framework.

As far as I can see no software source is available that simulates a single photon double slit experiment. This website demonstrates the difficulty most non-physicists have in trying to understand this experiment.

https://www.researchgate.net/post/Data_for_double_slit_experiment (https://www.researchgate.net/post/Data_for_double_slit_experiment)

 FYI I adhere to the school of thought that believes the universe is a type of computer. I think sometime in the next 100 years years we will be able to develop our own universes.
Title: Re: How is the quantum double slit experiment set up?
Post by: Bored chemist on 19/12/2018 10:36:16
To get to this conclusion, I have had to do an enormous amount of research. It seems it is something the physics community don't want the rest of the world to know.
If you have done "an enormous amount of research", how come it took you three attempts to find a (numbered) equation in a paper?

Also, given the "publish or perish" attitude of most academic organisations, how would it be possible that "the physics community don't want the rest of the world to know"?

Essentially all of their knowledge is published.
Title: Re: How is the quantum double slit experiment set up?
Post by: jeffreyH on 19/12/2018 14:26:33
The photon behaves like a particle and a wave. It doesn't need anyone's permission to do that. The pioneers of QM were simply trying to describe what they observed. They themselves were uncomfortable with the implications. This doesn't affect the photon in the slightest. It is simply that the universe works in ways we don't fully understand yet.
Title: Re: How is the quantum double slit experiment set up?
Post by: yor_on on 19/12/2018 16:40:37
It depends. One way of looking at it would be to say that the experimental setup defines the outcome. But if you want a definition allowing you to know the outcome every time in every experimental setup, beforehand, it becomes more difficult I think. That's what statistics is for, and probability. I don't really think you will be able to create it actually, if you could you would be ahead of current physics :) And possibly close to some theory of everything.
=

As for Carmen she struck me as being rather arrogant and high handed. She failed to define what she wanted and when people misunderstood her she got mad :) instead of trying to clear the misunderstanding. Normal people would tell her to f* off I suspect, if it happened in the street. Those guys didn't.
Title: Re: How is the quantum double slit experiment set up?
Post by: mxplxxx on 19/12/2018 20:38:27
As for Carmen she struck me as being rather arrogant and high handed. She failed to define what she wanted and when people misunderstood her she got mad :) instead of trying to clear the misunderstanding. Normal people would tell her to f* off I suspect, if it happened in the street. Those guys didn't.

Too true! Both sides communicated appallingly though. It illustrates the problem I have found most prevalent in forums where people veer away from answering the questions.
Title: Re: How is the quantum double slit experiment set up?
Post by: mxplxxx on 19/12/2018 20:45:04
From logic.
In the video you can see that an interference pattern is created- that clearly requires wave behaviour.
You can also see that the same pattern is generated when you count photons (as a function of position).
To get single photons (where only 1 is present in the apparatus at a time) and an interference pattern, requires the light to have both the properties of a wave and of  particle.

So, a whole branch of physics was started based on an educated guess and 100 years later noone can prove whether the guess was correct or not:).
Title: Re: How is the quantum double slit experiment set up?
Post by: Bored chemist on 19/12/2018 21:27:17
100 years ago, nobody could count photons.
But they came up with a theory.
That theory said that, if QM is real then, even if you reduce the power to the point where only 1 photon goes through the kit at a time, you will still get an interference pattern because photons are both particles and waves.

Many years later, when equipment had improved, we were able to count photons.
And the prediction was shown to be correct.
So have thousands of other things- from obscure experiments, to the workings of the computer you are using.

That's how science works. Nobody can ever prove that it's true- but, so far, nobody has shown that it's wrong.

Did you consider finding out how science works before trying to tell everyone that it's wrong?
Title: Re: How is the quantum double slit experiment set up?
Post by: Bored chemist on 19/12/2018 21:40:36

As for Carmen she struck me as being rather arrogant and high handed. She failed to define what she wanted and when people misunderstood her she got mad :) instead of trying to clear the misunderstanding. Normal people would tell her to f* off I suspect, if it happened in the street. Those guys didn't.
I think Carmen's biggest problem is saying "I need real data from a real experiment because I think the pattern is not as coincidental as QM states. "
How absurd is it to imagine that people wouldn't notice?
It is obviously ridiculous to assume that the experts in the field didn't set up the experiment properly, and get accurate measurements.
Title: Re: How is the quantum double slit experiment set up?
Post by: mxplxxx on 19/12/2018 22:46:04
Did you consider finding out how science works before trying to tell everyone that it's wrong?
Not wrong. Not right. Just unproven:).
Title: Re: How is the quantum double slit experiment set up?
Post by: yor_on on 20/12/2018 06:48:02
" So, a whole branch of physics was started based on an educated guess and 100 years later noone can prove whether the guess was correct or not:)."

It's not like that, one way of looking at it might be from a point of trial and error. With that comes new ideas and definitions. You find a experiment to give you some really weird results, that starts you thinking, and you search for a way to make it make sense. Later experiments may be constructed to test that idea. That's one of the most important things you need to do, to find that experiment testing your new thoughts. That way everything builds, there's a lot of new definitions today, that wouldn't have made sense to people a hundred and fifty years ago. You follow the way nature presents itself and then you search for how to explain it. It lead us to probabilities, lights duality, superpositions, relativity, conservation laws, etc etc.  You can't ignore nature.
=

Everything is a educated guess, isn't it? From you stepping out your door, expecting yourself to exist tomorrow too, to getting an idea of a new way to do something. There are no certainties in life, and modern physics is built to accommodate this.
Title: Re: How is the quantum double slit experiment set up?
Post by: yor_on on 20/12/2018 07:12:06
Hmm, seem I deleted the wrong comment? Or that the server did.

Anyway, I agree BC. also I think the way they accommodated her needs was from part compassion, part curiosity, to what she might have thought up. Not everyone is nice, doesn't mean they can't have original thinking though.
Title: e: How is the quantum double slit experiment set up?
Post by: mxplxxx on 20/12/2018 07:48:23
" So, a whole branch of physics was started based on an educated guess and 100 years later noone can prove whether the guess was correct or not:)."

It's not like that, one way of looking at it might be from a point of trial and error. With that comes new ideas and definitions. You find a experiment to give you some really weird results, that starts you thinking, and you search for a way to make it make sense. Later experiments may be constructed to test that idea. That's one of the most important things you need to do, to find that experiment testing your new thoughts. That way everything builds, there's a lot of new definitions today, that wouldn't have made sense to people a hundred and fifty years ago. You follow the way nature presents itself and then you search for how to explain it. It lead us to probabilities, lights duality, superpositions, relativity, conservation laws, etc etc.  You can't ignore nature.
=

Everything is a educated guess, isn't it? From you stepping out your door, expecting yourself to exist tomorrow too, to getting an idea of a new way to do something. There are no certainties in life, and modern physics is built to accommodate this.

Wave/particle duality as an idea is a different kettle of fish isn't it?. The idea that a quantum entity exists in a superposition of evolving states (btw a concept that physics has difficulty explaining) until it is observed when it, apparently magically, turns into a particle at a fixed place in space/time takes a great deal of accepting. Especially when the nature of an an observation is not well defined. One cannot help but think (as Einstein did) that another as yet unknown mechanism is at work. Personally I love the theory as it appears to make the universe a very personal place where your choices in life lead, incredibly, to very definite outcomes. This may be your personal universe and the rest of us are just players.
Title: Re: How is the quantum double slit experiment set up?
Post by: PmbPhy on 20/12/2018 16:07:11
The photon behaves like a particle and a wave.
All elementary particles, including photons, have wave properties depending on the experiment used detect then. A single photon passing through a double slit behaves like a particle. However the wave properties are apparent when one repeats the experiment a large number of times (either by using the same apparatus or having a multitude such apparatus. The repetition is called an ensemble (aka collection).

By the way. The wave of a proton depends of the wave function and varies the same way that one can't simply ask "What is the energy of a photon?"
Title: Re: How is the quantum double slit experiment set up?
Post by: mxplxxx on 20/12/2018 23:00:33
Actually, all photons have the same energy (Planck's constant joules), but different power. Energy comes in units called quanta - and this is the basis of quantum theory. A photon always contains one quantum of energy. A lower power photon will not knock an electron from a black body but a higher power one will. See  https://www.google.com/search?rlz=1C1CHBD_en-GBAU826AU826&ei=3xccXKyMJsSS9QOi2IfYAg&q=plancks+constant+energy+of+one+cycle&oq=plancks+constant+energy+of+one+cycle&gs_l=psy-ab.3...584984.596812..597188...0.0..0.415.9758.2-30j5j1....2..0....1..gws-wiz.......0j0i71j0i67j0i131j0i10j0i22i30j0i13j0i8i13i30j0i22i10i30j33i22i29i30j33i160.Q_EF2BHDRkU (https://www.google.com/search?rlz=1C1CHBD_en-GBAU826AU826&ei=3xccXKyMJsSS9QOi2IfYAg&q=plancks+constant+energy+of+one+cycle&oq=plancks+constant+energy+of+one+cycle&gs_l=psy-ab.3...584984.596812..597188...0.0..0.415.9758.2-30j5j1....2..0....1..gws-wiz.......0j0i71j0i67j0i131j0i10j0i22i30j0i13j0i8i13i30j0i22i10i30j33i22i29i30j33i160.Q_EF2BHDRkU)
Title: Re: How is the quantum double slit experiment set up?
Post by: alancalverd on 20/12/2018 23:20:19
Not true.

Planck's constant does not have the dimensions of energy.

Power is the rate of deposition or expenditure of energy. The power of a photon beam depends on the density of photons. The notion that a single photon has power is meaningless.

Energy is not necessarily quantised.

Different photons have different energies. A blue photon has about twice the energy of a red photon, which is why we see color, and an x-ray photon has about 10,000 times as much energy, which is why we can photograph the insides of things.

"Knocking an electron from a black  body" is a meaningless collection of pseudoscientific words.

If you want to sing opera, it's a good idea to learn some Italian and stick to the music. If you want to discuss physics it's a good idea to learn the language and respect the dimensions.
Title: Re: How is the quantum double slit experiment set up?
Post by: Bored chemist on 20/12/2018 23:37:22
Did you consider finding out how science works before trying to tell everyone that it's wrong?
Not wrong. Not right. Just unproven:).
The way science works it to accept that most things are unproven. But what's important is that you can at least discard the stuff that's disproven.

That policy has, in fact, been shown to work- for example- you computer works. Planes fly medicines cure disease.

Discounting that sort of evidence just makes you look silly.
Title: Re: How is the quantum double slit experiment set up?
Post by: Bored chemist on 20/12/2018 23:39:25
Actually, all photons have the same energy (Planck's constant joules), but different power. Energy comes in units called quanta - and this is the basis of quantum theory.
Like I said, it would be a good idea to learn some science before trying to criticise it.
Hint; red photons have roughly half the energy of violet one.
Title: Re: How is the quantum double slit experiment set up?
Post by: Bored chemist on 20/12/2018 23:40:25
I have had to do an enormous amount of research.
What research did you do that left you so ignorant of fairly basic science?
Title: Re: How is the quantum double slit experiment set up?
Post by: PmbPhy on 21/12/2018 01:44:17
Actually, all photons have the same energy (Planck's constant joules), but different power. Energy comes in units called quanta - and this is the basis of quantum theory.
That is incorrect. Different photons can have different energy. If they were all the same energy then the rainbow would not be colorful. Watching a movie means seeing different colors. Different colors means different photon energies. Where did you get idea otherwise?

Energy is not always quantized in QM. For a the energy of a particle to be quantized it must be in potential well.

You should study physics in more detail since your basic understanding is quite wrong. I recommend picking learning math and physics from college level texts. They usually explain the physics/math quite well.

By the way. Planck's constant is not in units of energy. It's in units of action = [Joules-sec].
Title: Re: How is the quantum double slit experiment set up?
Post by: mxplxxx on 21/12/2018 01:48:54
Higher power photons have shorter wavelengths than lower power photons. This means they transfer their energy quicker. But the total amount of energy transferred is the same for all. You can fire a beam of low power photons at (say) a piece of aluminium and it will not knock any electrons from the aluminium irrespective of how intense the photon beam is. The higher power photons will knock electrons from the aluminium irrespective of the intensity of the photon beam. This fact was discovered by Einstein in his basic quantum research. See https://www.pitt.edu/~jdnorton/teaching/HPS_0410/chapters/quantum_theory_origins/ (https://www.pitt.edu/~jdnorton/teaching/HPS_0410/chapters/quantum_theory_origins/) Photoelectric Effect.

See also https://www.researchgate.net/publication/325462944_Planck's_Constant_and_the_Nature_of_Light.
 (https://www.researchgate.net/publication/325462944_Planck's_Constant_and_the_Nature_of_Light.).

Note also that any discussion of photons in quantum theory is hindered by the fact that we don't know what the wave function of a photon is. We are totally reliant on the classical description of light as a transverse electromagnetic wave.
Title: Re: How is the quantum double slit experiment set up?
Post by: PmbPhy on 21/12/2018 02:02:51
Higher power photons have shorter wavelengths than lower power photons.
You're wrong. It's a physical fact that different photons have can have different energies. Its meaningless to speak of a photon having a certain amount of power. Do you know what power is? Its the energy of a system divided by the amount the energy changes over a period of time. I.e. P = dE/dt

Title: Re: How is the quantum double slit experiment set up?
Post by: mxplxxx on 21/12/2018 02:03:06
"Knocking an electron from a black  body" is a meaningless collection of pseudoscientific words.
True! I use aluminium as an example in my latest post.
Title: Re: How is the quantum double slit experiment set up?
Post by: PmbPhy on 21/12/2018 02:47:16
Higher power photons have shorter wavelengths than lower power photons. This means they transfer their energy quicker.
Wherever you heard/learned that, you're till wrong and you're refusing to state where you learned it so this is my last post in this thread until you do.

I know the principles of quantum mechanics as well as any physicist and that's from advanced QM texts all of which agree all which confirm that different photons can have different energies and that the power of a photon is meaningless.

And there is a wave function of a photon which shows you're assumption otherwise to be wrong. Different photon possibly different wave function.

Later gator.
Title: Re: How is the quantum double slit experiment set up?
Post by: mxplxxx on 21/12/2018 03:49:51
You're wrong. It's a physical fact that different photons have can have different energies. Its meaningless to speak of a photon having a certain amount of power. Do you know what power is? Its the energy of a system divided by the amount the energy changes over a period of time. I.e. P = dE/dt
Sure I know what power is. I also know what energy is. In joules, it is "equal to the work done by a force of one newton when its point of application moves one metre in the direction of action of the force". No time involved in this. But time is involved in probably the most basic equation of quantum physics E= hf which gives the energy of a particle that will be transferred in 1 second. This is a power unit. The energy that will be transferred in one cycle is E/f which is h which is always the same for all particles. People confuse energy and power and this seems to have happened sometime in the past with physics (unless I am horribly wrong - always a possibility:).
Title: Re: How is the quantum double slit experiment set up?
Post by: Bored chemist on 21/12/2018 10:05:32
unless I am horribly wrong

You are.
Several of us have pointed this out.
You are refusing to listen.

What are you doing here if you willfully decline to learn?
Title: Re: How is the quantum double slit experiment set up?
Post by: mxplxxx on 22/12/2018 05:03:04
Several of us have pointed this out.
I need understanding, not directives. What is the underlying meaning of the E=hf equation? I have searched high and low for this to no avail. It seems its meaning has been lost in the sands of time. If any of you can provide a lucid explanation of the equation, I will take my hat off to you:).
Title: Re: How is the quantum double slit experiment set up?
Post by: Bored chemist on 22/12/2018 09:56:16
have searched high and low for this to no avail.
OK, So, if you googled it, you probably got the wiki page
https://en.wikipedia.org/wiki/Photon_energy

Which bits of it  do we need to explain?

Also, how did you come to the conclusion that an equation that gives 400 million his on google is " lost in the sands of time. "?
Title: Re: How is the quantum double slit experiment set up?
Post by: mxplxxx on 22/12/2018 12:44:21
have searched high and low for this to no avail.
OK, So, if you googled it, you probably got the wiki page
https://en.wikipedia.org/wiki/Photon_energy

Which bits of it  do we need to explain?

Also, how did you come to the conclusion that an equation that gives 400 million his on google is " lost in the sands of time. "?

Th Wiki page just repeats what countless others on the Internet say about the equation. They do not  discuss the meaning of the the equation and the concepts behind it and the implications for quantum theory. THey in particular do not discuss what the energy of a photon actually means. So Energy = a constant divided by the frequency of the photon. How does this come about? What is the frequency of a photon, especially given the wave function of a photon is not known. If it is the frequency of an EM wave, how does this apply to a quantum entity. The whole thing is a shocking mess. I am a systems analyst as well as a programmer and I can't work it out. And I am as smart as smart as can be with Uni maths as well.

Lost in the sands of time  ... The meaning of Action seems to have undergone several metamorphoses and I cannot work out what it means today.
Title: Re: How is the quantum double slit experiment set up?
Post by: Bored chemist on 22/12/2018 14:01:13
Energy = a constant divided by the frequency of the photon.
It's not divided by frequency, it is multiplied.
h Wiki page just repeats what countless others on the Internet say about the equation.
Well, it's probably correct.
They do not  discuss the meaning of the the equation
OK, what it says is "Photon energy is the energy carried by a single photon. "
Which words don't you understand?
Did you try googling them?
especially given the wave function of a photon is not known.
It is known.
It's just that you can't understand it.
And I am as smart as smart as can be with Uni maths as well.
It took you several tries to find a numbered equation in some text.
And now you are pretending that it doesn't exist.

How smart do you think that makes you look?

May I make a suggestion?
I'm going to be away from home a bit over the Xmas/ New year period, so why don't you take the chance to actually study this stuff.
You might start here.
https://www.khanacademy.org/science/physics/quantum-physics/modal/v/photon-energy
Title: Re: How is the quantum double slit experiment set up?
Post by: jeffreyH on 22/12/2018 19:46:11
Higher power photons have shorter wavelengths than lower power photons. This means they transfer their energy quicker. But the total amount of energy transferred is the same for all. You can fire a beam of low power photons at (say) a piece of aluminium and it will not knock any electrons from the aluminium irrespective of how intense the photon beam is. The higher power photons will knock electrons from the aluminium irrespective of the intensity of the photon beam. This fact was discovered by Einstein in his basic quantum research. See https://www.pitt.edu/~jdnorton/teaching/HPS_0410/chapters/quantum_theory_origins/ (https://www.pitt.edu/~jdnorton/teaching/HPS_0410/chapters/quantum_theory_origins/) Photoelectric Effect.

See also https://www.researchgate.net/publication/325462944_Planck's_Constant_and_the_Nature_of_Light.
 (https://www.researchgate.net/publication/325462944_Planck's_Constant_and_the_Nature_of_Light.).

Note also that any discussion of photons in quantum theory is hindered by the fact that we don't know what the wave function of a photon is. We are totally reliant on the classical description of light as a transverse electromagnetic wave.

The energy of all photons is not the same. The quantum of action (h) is useless without the wavelength. The Planck constant is also not time dependent. Which it would have to be for your argument to be correct. Go and read a biography of Max Planck. I am currently reading "No time to be brief" the scientific biography of Wolfgang Pauli. You will find such study more rewarding than posting aimlessly on forums. Then you could come back and have a constructive debate.
Title: Re: How is the quantum double slit experiment set up?
Post by: yor_on on 22/12/2018 20:43:02
Mxp

Don't let it rest too long, use the Internet and your mind. Physics is in someways a very personal thing, make it your own. Pmb (Pete) knows what he's talking about.
Title: Re: How is the quantum double slit experiment set up?
Post by: PmbPhy on 22/12/2018 21:27:12
Higher power photons have shorter wavelengths than lower power photons. This means they transfer their energy quicker. But the total amount of energy transferred is the same for all. You can fire a beam of low power photons at (say) a piece of aluminium and it will not knock any electrons from the aluminium irrespective of how intense the photon beam is. The higher power photons will knock electrons from the aluminium irrespective of the intensity of the photon beam. This fact was discovered by Einstein in his basic quantum research. ...
It's clear that he wants to be spoon fed rather then doing the work himself. Otherwise he could pick up a text such as the excellent one by French and Taylor. In that text, as in all other QM texts, the authors explain how Planck came to find E = hf. This equation quite literally means that the frequency of a photon, f, is related to its (kinetic) energy by E = hf where h = Planck's constant. If a particle has a non-zero proper mass but has momentum p and wavelength L then L = h/p, wichh also holds for photons => Different momentum -> different frequency

These have been experimentally verified. He should go to the new theory forum if he wants to prove otherwise. He's claiming something is true which cannot be experimentally verified and therefore has no place in physics. That's why I gave up on him, i.e. I don't do "new theories" since in the 20 years I've been posting I've never seen one which is reasonable. They all claim that current theory is wrong or incomplete but do not provide solid reasoning to explain the nonsense they post.
Title: Re: How is the quantum double slit experiment set up?
Post by: mxplxxx on 23/12/2018 10:36:29
OK, I have made a big mistake. Many humble apologies for wasting your times. The current theory works just fine. It is obvious when you fiddle around the the E=hf equation that it does exactly what physics says it does.

h = 7
 therefore
E*t = 7
 
t = 1/f
therefore
E*(1/f)= 7
therefore
E= 7f
therefore
E= hf

and, therefore

E = energy of a photon = the energy of one oscillation of the photon.
Title: Re: How is the quantum double slit experiment set up?
Post by: Colin2B on 24/12/2018 07:22:11
OK, I have made a big mistake. ........ the E=hf equation that it does exactly what physics says it does.
You are right about that

But totally wrong about this:
h = 7
 therefore
E*t = 7
 
t = 1/f
therefore
E*(1/f)= 7
therefore
E= 7f
therefore
E= hf

and, therefore

E = energy of a photon = the energy of one oscillation of the photon.
You claim to know maths, but get the simplest things wrong.
This is garbage.
Title: Re: How is the quantum double slit experiment set up?
Post by: mxplxxx on 24/12/2018 07:40:42
This is garbage.
Why?
Title: Re: How is the quantum double slit experiment set up?
Post by: Bored chemist on 24/12/2018 12:43:35
For  a start h really really isn't 7.
Also there's no explanation of how you even got to this "E = energy of a photon = the energy of one oscillation of the photon."
which is utterly wrong
Title: Re: How is the quantum double slit experiment set up?
Post by: opportunity on 24/12/2018 12:50:29
Energy is many things derived from.

Usually we have quantum jumps of electrons.

Unusually we explode things to create theory.....and then put that in, lock that in......as a process of discovery.
Title: Re: How is the quantum double slit experiment set up?
Post by: yor_on on 24/12/2018 13:55:13
Now I'm guessing Mxp. What I'm guessing is your foundation of physics to be more wobbling than your ability to program. That's no fault, it's just a matter of persistence. That's why you will need to check it up. To see how probability and Heisenberg's Uncertainty Principle define physics today. It's not simple, and it take you years to get a grasp of how people think about it. Even those highly educated from the beginning will hesitate when it comes to take the step from being very good at, let's call it 'civilengineering' here in Sweden, to take that step to becoming a physicist and researcher. It takes another amount of years and the personal benefits of it is very unclear. Don't take the criticism too hard, go back to the history of physics and try follow its timeline. That will help you see how it works.

And a merry Xmas to you and us all.
Title: Re: How is the quantum double slit experiment set up?
Post by: Bill S on 24/12/2018 15:47:03
Quote from: Mxplxxx
OK, I have made a big mistake. Many humble apologies for wasting your times.

Possibly you didn’t get what you needed from this thread, but don’t assume you wasted everyone’s time.  I’ve not had time to read it all yet, or to follow the links, but I have already learned a few things.

As a non-scientist/mathematician, I frequently struggle with answers to my questions.  I empathise with your feeling that sometimes the experts answer questions that differ from those asked. 

I’m not equating my “hitch-hiker” position with yours, but I reason that if the experts have the patience to respond to my questions, I should have the patience to accept that they don’t always appreciate the naivety of those questions. 
Title: Re: How is the quantum double slit experiment set up?
Post by: mxplxxx on 24/12/2018 22:05:43
For  a start h really really isn't 7.
Obviously.
Title: Re: How is the quantum double slit experiment set up?
Post by: mxplxxx on 26/12/2018 05:31:44
These have been experimentally verified. He should go to the new theory forum if he wants to prove otherwise. He's claiming something is true which cannot be experimentally verified and therefore has no place in physics.
This is too simplistic an answer. Physics consists of theoretical and experimental branches. See https://www.youtube.com/watch?v=HN83u9Cpa9k (https://www.youtube.com/watch?v=HN83u9Cpa9k). There are many theories in physics that have not been experimentally verified. Pretty much all of String Theory for example is unverified (https://www.forbes.com/sites/startswithabang/2015/12/23/why-string-theory-is-not-science/#5f2c1f236524 (https://www.forbes.com/sites/startswithabang/2015/12/23/why-string-theory-is-not-science/#5f2c1f236524)). As a lay person, what is hard science and what is speculation in Physics is often difficult to distinguish.
Title: Re: How is the quantum double slit experiment set up?
Post by: Bored chemist on 26/12/2018 13:27:03
As a lay person,
Then learn enough to stop being a layman.
Title: Re: How is the quantum double slit experiment set up?
Post by: mxplxxx on 28/12/2018 00:43:22
It would seem (shockingly!) that E=hf relates only to Electromagnetic waves, not photons (https://physics.stackexchange.com/questions/90646/what-is-the-relation-between-electromagnetic-wave-and-photon).
Title: Re: How is the quantum double slit experiment set up?
Post by: yor_on on 28/12/2018 09:34:54
That was a nice link Mxp, but I think that equation is about the energy of a particle, as a electron. " The h is Planck's constant, which corresponds to 1 quanta of energy, which is present in energy packets. The f is the frequency of the particle, and when you multiply the two together, you get the energy. This equation is used in basic quantum mechanics to derive equations based on the quantum energy states of electrons around the nucleus of an atom. "

The duality of light is a proved fact, nowadays defined as a 'excitation in a field'. Any model you find is just a model, the best physics can make at the current stage. Thinking of it as a field makes sense from a lot of perspectives, and thinking of a photon as a 'wavepacket' is closely related to your comment on E=hf.

But they are all models, not written in stone. Physics can't pre-define nature, not yet anyway, as far as I know :)
I agree with you, starting with the duality makes for a very good dive into physics.

https://www.quora.com/What-is-behind-E-hf-of-a-photon?

Title: Re: How is the quantum double slit experiment set up?
Post by: mxplxxx on 29/12/2018 03:15:43
That was a nice link Mxp, but I think that equation is about the energy of a particle, as a electron
Lots and lots of apparent contradictions and anomalies in quantum theory. The following is  a summary that has been gleaned from my extensive travels in trying to understand quantum physics and is backed up by the discussions in the following link (https://physics.stackexchange.com/questions/90646/what-is-the-relation-between-electromagnetic-wave-and-photon).

E=hf relates to the energy of an electromagnetic wave, not a photon. This is a classical equation, not its equivalent quantum wave function. Because light is a EM wave, physics has taken the E-hf equation and applied it to get the energy of a photon, apparently without scientific evidence to back up the theory. The wave function of a photon is not known.

Apparently, the results in double slit experiment for a single photon cannot be repeated using the E=hf equation. You need the wave function for this and this is not known for a photon.

To make matters worse, an electromagnetic wave is actually two waves that operate transversely and at right angles to one another. This seems to involve 3 energies, the forward direction energy, the sideways electro wave energy and the sideways  magnetic wave energy. The implications of this for a photon seem to have been overlooked by the physics community.

I photon is supposed to have no mass. Where then does its momentum come from and what is the equation for the momentum?

The fact that an EM waver travels at the speed of light means that its transverse component exceeds the speed of light. This anomaly is not explained in physics.

What is the difference between an EM wave, a probability wave and a Wave function for a photon?

Red shift etc. seem to involve the photon losing energy, in contradiction of Newtons conservation of energy law.

Finally, it has to be noted that a photon can have any energy that is a multiple of h. This makes it not one type of particle but many. This fact is almost universally overlooked by the physics community.

Title: Re: How is the quantum double slit experiment set up?
Post by: Bored chemist on 29/12/2018 11:47:01
E=hf relates to the energy of an electromagnetic wave, not a photon.
Says who?
The speed of light is constant so frequency and wavelength are fundamentally linked- if you divide C by one, you get the other.

Einstein's classic work on the photoelectric effect was done with monochromatic light which had been "filtered" through a diffraction grating to select the wavelength (and thus, the frequency).
So we have photons, of a known frequency- measured by the monochromator giving electrons of known energy- measured by the stopping potential.

And, of course, they follow the  E=hf relationship.

The experiment works.
Only you see it as a contradiction.

Apparently, the results in double slit experiment for a single cannot be repeated using the E=hf equation.
Sorry, would you like to rewrite that? It doesn't make sense.
This seems to involve 3 energies, the forward direction energy, the sideways electro wave energy and the sideways  magnetic wave energy. The implications of this for a photon seem to have been overlooked by the physics community.

The interchange of energy between electric and magnetic fields  poses no more problems for physics then the exchange of potential and kinetic energy of a simple pendulum.

Again, it seem only you see it as difficult.

I photon is supposed to have no mass
No, it has no rest mass.
Where then does its momentum come from
Relativistic mass.
what is the equation for the momentum?
It was sorted out in 1924. Why are you not aware of it?
https://en.wikipedia.org/wiki/Matter_wave
Title: Re: How is the quantum double slit experiment set up?
Post by: yor_on on 29/12/2018 13:57:40
" the results in double slit experiment for a single cannot be repeated using the E=hf equation "

Why?
Title: Re: How is the quantum double slit experiment set up?
Post by: mxplxxx on 29/12/2018 22:56:36
Quote from: mxplxxx on Today at 03:15:43
Where then does its momentum come from
Relativistic mass.
Highly controversial. http://math.ucr.edu/home/baez/physics/ParticleAndNuclear/photon_mass.html
Title: Re: How is the quantum double slit experiment set up?
Post by: mxplxxx on 30/12/2018 05:22:10
Of course the biggest problem with the dual slit experiment is the quantum nature of light. How can a supposedly indivisible photon split into two?
Title: Re: How is the quantum double slit experiment set up?
Post by: mxplxxx on 30/12/2018 06:11:19
Quote from: mxplxxx on Yesterday at 03:15:43
Apparently, the results in double slit experiment for a single cannot be repeated using the E=hf equation.
Sorry, would you like to rewrite that? It doesn't make sense
The double slit experiment has the photon existing as a wave prior to passing through the slits. Physics generally equates the wave nature of a photon with a an EM wave. But what goes through the double slit seems to be a probability wave, not an EM wave. Is it possible a photon exists in TWO wave states?
Title: Re: How is the quantum double slit experiment set up?
Post by: yor_on on 30/12/2018 08:04:51
First of all. That a 'photon' can  'split' is a example of the particle wave duality. The only way you can question this is by not accepting the premise that it has one. You more or less correctly describe it in your last post " The double slit experiment has the photon existing as a wave prior to passing through the slits. ". More or less as nobody can tell you 'how' and 'when' it 'decide to do so.

That's what indirect experiments are all about, to prove f.ex. a 'propagation', or  whether the 'photon' can 'know' beforehand what obstacles a experimenter sets up for it etc etc. Indirect evidence is the nature of things in quantum physics as any 'nudge' of that photon will define it as a particle (or wave) with a defined polarization/spin etc. Personally I have a certain reluctance when it comes to accept indirect evidence as being 'the whole truth and so help me God', but that's me :)

As you say a 'photon' is by its nature defined to be indivisible, in the same manner as we define a electron to be so. Often called the smallest measurable building blocks of nature. But it's not the whole story and electrons seems to have their own ways too. https://en.m.wikipedia.org/wiki/Orbiton

Those results do point to a 'field' existing in my eyes, or something interconnected anyway.
Title: Re: How is the quantum double slit experiment set up?
Post by: Bored chemist on 30/12/2018 13:18:20
Quote from: mxplxxx on Today at 03:15:43
Where then does its momentum come from
Relativistic mass.
Highly controversial. http://math.ucr.edu/home/baez/physics/ParticleAndNuclear/photon_mass.html
Not controversial at all.
The page you site says "In classical electromagnetic theory, light turns out to have energy E and momentum p, and these happen to be related by E = pc.  Quantum mechanics introduces the idea that light can be viewed as a collection of "particles": photons.  Even though these photons cannot be brought to rest, and so the idea of rest mass doesn't really apply to them, we can certainly bring these "particles" of light into the fold of equation (1) by just considering them to have no rest mass.  That way, equation (1) gives the correct expression for light, E = pc, and no harm has been done.  Equation (1) is now able to be applied to particles of matter and "particles" of light.  It can now be used as a fully general equation, and that makes it very useful."

Of course the biggest problem with the dual slit experiment is the quantum nature of light. How can a supposedly indivisible photon split into two?
OK, for a start, they are not indivisible- you made that up.

For an encore, If I want to block up both slits in a typical "double slits" experiment, I can cover them with my finger.
I do not need to cut my finger in two to do this.

Photons are not "points".
Title: Re: How is the quantum double slit experiment set up?
Post by: mxplxxx on 31/12/2018 01:39:21
First of all. That a 'photon' can  'split' is a example of the particle wave duality. The only way you can question this is by not accepting the premise that it has one. You more or less correctly describe it in your last post " The double slit experiment has the photon existing as a wave prior to passing through the slits. ". More or less as nobody can tell you 'how' and 'when' it 'decide to do so.

That's what indirect experiments are all about, to prove f.ex. a 'propagation', or  whether the 'photon' can 'know' beforehand what obstacles a experimenter sets up for it etc etc. Indirect evidence is the nature of things in quantum physics as any 'nudge' of that photon will define it as a particle (or wave) with a defined polarization/spin etc. Personally I have a certain reluctance when it comes to accept indirect evidence as being 'the whole truth and so help me God', but that's me :)

As you say a 'photon' is by its nature defined to be indivisible, in the same manner as we define a electron to be so. Often called the smallest measurable building blocks of nature. But it's not the whole story and electrons seems to have their own ways too. https://en.m.wikipedia.org/wiki/Orbiton

Those results do point to a 'field' existing in my eyes, or something interconnected anyway.
Interesting thoughts. You have a refreshingly resilient attitude to physics. You seem to rely a lot on your intuition which, IMHO, you really need when doing research.
Title: Re: How is the quantum double slit experiment set up?
Post by: mxplxxx on 31/12/2018 02:33:28
It also occurs to me that the wavelength required to fit between the slits of the double slit experiment is that of the EM wave associated with the photon. But the waves the other side of the slits are possibility waves. What does this even mean?
Title: Re: How is the quantum double slit experiment set up?
Post by: yor_on on 31/12/2018 04:33:49
The interference pattern will be there, although less clearly, if you increase the separation between slits, so that doesn't change it. And I wouldn't call the pattern probabilistic, it should be consistent if you repeat a equivalent experiment. There is a small chance, and possibility, of some interference, of that interference pattern I mean :) but any probability for that has to be  v e r y  s m a l l, as I see it,, presuming the experiment to be consistently set up and repeated..

And no, and yes. I prefer to call it logic myself :) But as you say, some things are what you call, a intuition. We take too much for granted.
Title: Re: How is the quantum double slit experiment set up?
Post by: opportunity on 31/12/2018 04:49:54
To demonstrate the idea of a wave as light, take a light point source with waves radiating circumferentially from that point source. It goes out everywhere as a spherical front, right? Its fairly problematic to do that with particle beams everywhere radially.

So, to demonstrate the wave theory, the wider the slit, the greater the wave radiation beyond that slit, right?


Two slits....wave interference.
Title: Re: How is the quantum double slit experiment set up?
Post by: opportunity on 31/12/2018 04:54:20
For nature to design such a mechanism of light is profound....like an after-thought of the universe.
Title: Re: How is the quantum double slit experiment set up?
Post by: mxplxxx on 31/12/2018 05:47:49
For nature to design such a mechanism of light is profound....like an after-thought of the universe.
My take on the universe is that it is an evolving entity, constantly getting better until it "runs out of puff" (entropy-wise) at which point it collapses in on itself only to reappear at some point in the future as a big-bang that retains some of the features of the previous evolution.

The probability waves of the double slit experiment seem to help evolution. What is expected to occur does not always happen and the unexpected happenings can change an entity either for the worse, thus decreasing its chances of success in the future or for the better, thus increasing its chances of success in the future.

A successful entity in the universe is one that has mechanisms that help it survive in the face of constant uncertainty and "mistakes". Successful in the sense that it survives long enough to make sure it has offspring. Everything dies eventually, possibly running out of energy with which to correct mistakes.

I think the universe is trying to escape eternity which is a millstone around the neck of time-based entities.

IMHO, everything has a purpose. I see the The purpose of light being to carry change in state information from one particle to another. It may do this randomly ("pull") or by design ("push")  just as the neurotransmitters in the brain do (giving rise to the possibility that electrons may be light receptors).

Title: Re: How is the quantum double slit experiment set up?
Post by: opportunity on 31/12/2018 05:51:35
Why not? If you could see yourself in a house of mirrors you want just one thing, not endless reflections.
Title: Re: How is the quantum double slit experiment set up?
Post by: mxplxxx on 31/12/2018 06:47:51
I am a great believer in the universe as a computer that simulates itself. The double slit experiment likely accesses vast amounts of statistical information that is stored and maintained by something that approximates a computer.
Title: Re: How is the quantum double slit experiment set up?
Post by: mxplxxx on 31/12/2018 06:51:27
The fact that an EM waver travels at the speed of light means that its transverse component exceeds the speed of light. This anomaly is not explained in physics.
But has me wondering whether EM waves originate in the future (think tachyon https://www.scientificamerican.com/article/what-is-known-about-tachy/) or have a future component.
Title: Re: How is the quantum double slit experiment set up?
Post by: opportunity on 31/12/2018 06:54:58
If what happened before is a part of what must happen in the future, and light dances in between, that's almost steady-state.

If light happens before where it should come from, that's almost mysticism, right?
Title: Re: How is the quantum double slit experiment set up?
Post by: mxplxxx on 31/12/2018 07:10:35
If light happens before where it should come from, that's almost mysticism, right?
Isn't the collapse of the wavefunction actually mysticism? :)
Title: Re: How is the quantum double slit experiment set up?
Post by: mxplxxx on 31/12/2018 07:19:07
Modern philosophy with the idea:
Black and blue
And who knows which is which and who is who
Up and down
But in the end it's only round and round
Title: Re: How is the quantum double slit experiment set up?
Post by: opportunity on 31/12/2018 07:21:03
If light happens before where it should come from, that's almost mysticism, right?
Isn't the collapse of the wavefunction actually mysticism? :)

https://arxiv.org/pdf/1002.4568v1.pdf

I've a better explanation. Haven't posted it yet, formally.
No. It depends on distance, and for the lay man ultimately Q-E.

Title: Re: How is the quantum double slit experiment set up?
Post by: Bored chemist on 31/12/2018 12:25:59
My take on the universe is that it is an evolving entity, constantly getting better until it "runs out of puff" (entropy-wise)
Do you mean " constantly getting better" or do you mean " it "runs out of puff" (entropy-wise)"?
You can't have both.
Title: Re: How is the quantum double slit experiment set up?
Post by: yor_on on 31/12/2018 13:40:41
" The double slit experiment likely accesses vast amounts of statistical information that is stored and maintained by something that approximates a computer. " Maybe?

It's about HUP and our equivalent 'free will' that one, to me.

From your text I would guess you would like 'free will' to define that statistic, as in defining outcomes, by the way we measure and set up our experiment?. But think of a quantum computer using superpositions. To it all possibilities are there already, existing in those superpositions. So in what way should we define that 'storage'?
=

A 'many worlds' explanation would be that all possibilities are taken, although, depending on probability (statistics for how nature 'works') only choosing one (highest probability normally) for us doing the experiment. I don't like that one by several reasons, one being the one in where it seems centered on us, we become a 'privileged frame' in it to my eyes.

Another way that I like better, is the one in where the question becomes how this 'storage' function. One way to see it would be from something very complicated, meaning that everything is super positioned until 'measured', to something 'simple', in this case meaning a 'outcome'.

The whole idea of calling something 'complicated' is to me centered on our preconceptions. From the view of 'nature' maybe a superposition is what is 'simple'?
Title: Re: How is the quantum double slit experiment set up?
Post by: opportunity on 31/12/2018 15:04:55
You guys can fight that out, right?
Title: Re: How is the quantum double slit experiment set up?
Post by: Bored chemist on 31/12/2018 15:11:56
You guys can fight that out, right?
For the most part, the "fight" happened 70 years ago.
Evidence won.
Title: Re: How is the quantum double slit experiment set up?
Post by: Bored chemist on 31/12/2018 15:22:38
30 years ago I believed it.
What went wrong?
Did you forget?
Title: Re: How is the quantum double slit experiment set up?
Post by: mxplxxx on 01/01/2019 00:00:53
My take on the universe is that it is an evolving entity, constantly getting better until it "runs out of puff" (entropy-wise)
Do you mean " constantly getting better" or do you mean " it "runs out of puff" (entropy-wise)"?
You can't have both.
It basically eventually  "dies", but in the process accumulates a lifetime of experience which is available for the next revolution (aka Matrix:)). We seem to be at a nadir of entropy currently where things are very stable (comparatively) and plenty of energy is available for evolution.
Title: Re: How is the quantum double slit experiment set up?
Post by: mxplxxx on 01/01/2019 00:23:03
" The double slit experiment likely accesses vast amounts of statistical information that is stored and maintained by something that approximates a computer. " Maybe?
I see the universe as a hierarchical state machine (HSM) which is easily programmed on a computer. It is one of my specialties. In a HSM all possible states are present but they can be "condensed" via abstraction.  In this scheme of things, you and I are abstractions except (possibly) for our consciousness. A superposition of states may not be as taxing to the universe as you would imagine. Plus the collapse of the wave function is easily achieved. 

Title: Re: How is the quantum double slit experiment set up?
Post by: Bored chemist on 01/01/2019 12:11:16
It basically eventually  "dies", but in the process accumulates a lifetime of experience which is available for the next revolution (aka Matrix:)).
"The Matrix" isn't real.
Entropy will be at a maximum at the "death" of the universe. It will be totally disorganised and will carry zero information.


Stop posting + go + learn.
Title: Re: How is the quantum double slit experiment set up?
Post by: mxplxxx on 01/01/2019 22:10:34
Entropy will be at a maximum at the "death" of the universe. It will be totally disorganised and will carry zero information.
Depends on which of a multitude of Universe end scenarios has occurred. From http://www.bbc.com/earth/story/20150602-how-will-the-universe-end:

"Inflation has an intriguing consequence for the ultimate fate of the universe. The theory dictates that the universe we inhabit is just one small part of a multiverse, with an eternally inflating background continually spawning "pocket universes" like our own.
"If that's the case, even if we're convinced that an individual pocket universe will ultimately die through refrigeration, the multiverse as a whole will go on living forever, with new life being created in each pocket universe as it's created," says Guth. "In this picture, the multiverse as a whole is genuinely eternal, at least eternal into the future, even as individual pocket universes live and die.""

Also, Hawking has postulated that Black Holes are portals to other universes It is possible information about the current universe could be stored in one of these universes and used when recreating the current universe.

Title: Re: How is the quantum double slit experiment set up?
Post by: Bored chemist on 02/01/2019 13:42:47
even if we're convinced that an individual pocket universe will ultimately die through refrigeration
So, we die from entropy and all the information in this universe is lost.
We can not (by definition) know what happens in other universes.
Title: Re: How is the quantum double slit experiment set up?
Post by: mxplxxx on 03/01/2019 02:46:35
It is clear quantum physics is in a mess. A large number of competing theories exist and some parts of quantum physics approach mysticism. Very little consensus exists. See https://1drv.ms/u/s!AkkAzGDByUeBl8VPLqTemQdutRVAcQ (https://1drv.ms/u/s!AkkAzGDByUeBl8VPLqTemQdutRVAcQ)

Large A proposed solution to the double slit experiment using Feynman Path Integral formalism makes much more sense than a probability wave. https://phys.org/news/2014-10-superposition-revisited-resolution-double-slit-paradox.html (https://phys.org/news/2014-10-superposition-revisited-resolution-double-slit-paradox.html).

Also the Superposition of States (sos) theory is just a steal from finite state machine (fsm) software systems fsm handles multiple concurrent states no worries, and a single event (aka an observation) can reduce the concurrent states to a single state. No need for a collapse of a wave function.This event would typically be a user click and in between click the software can evolve to its hearts content. It is all in the programming and the universe would have no trouble at all doing it.  Heaven knows why sos seems to have arisen from the double slit experiment.
Title: Re: How is the quantum double slit experiment set up?
Post by: opportunity on 03/01/2019 08:11:14
I think it would be a handy exercise to list all the great discoveries independent of stellar phenomena, and those discoveries lab-born entirely relevant to stellar phenomena, and thirdly theories of stellar phenomena we cannot yet prove in the lab.
Title: Re: How is the quantum double slit experiment set up?
Post by: Bored chemist on 03/01/2019 14:13:37
I think it would be a handy exercise to list all the great discoveries independent of stellar phenomena, and those discoveries lab-born entirely relevant to stellar phenomena, and thirdly theories of stellar phenomena we cannot yet prove in the lab.
Feel free.
You can list the members of the 1966 world cup squad too if you like.
Title: Re: How is the quantum double slit experiment set up?
Post by: mxplxxx on 04/01/2019 10:52:04
I think it would be a handy exercise to list all the great discoveries independent of stellar phenomena, and those discoveries lab-born entirely relevant to stellar phenomena, and thirdly theories of stellar phenomena we cannot yet prove in the lab
Interesting. My Hierarchical Abstractions Framework software contains objects (particles) and Systems. At the center of each system is an object I call a Star. Actually, it is the "star of the show" but would be a real star/sun in a solar system. The universe, it would seem, contains a hierarchy of systems consisting of a central object  and peripheral sub-systems. All systems from the lowest particle to the universe itself are types of this basic system. The center of a central object is very likely the consciousness of the system and is often a black hole.
Title: Re: How is the quantum double slit experiment set up?
Post by: Bored chemist on 05/01/2019 13:06:19
The center of a central object is very likely the consciousness of the system
In what way?
Title: Re: How is the quantum double slit experiment set up?
Post by: mxplxxx on 05/01/2019 13:23:01
The center of a central object is very likely the consciousness of the system
In what way?
The center of a system is where the top-level state of a system resides. It is where imperative events from the system's parent are processed, where imperative events are sent to the system's siblings and children,  and where change of state events that are received from the system's siblings and children are processed. Just like your own consciousness (where parent equates to your higher self). A black hole is a likely candidate for consciousness because it can survive death and is outside of time (both guesses).
Title: Re: How is the quantum double slit experiment set up?
Post by: Bored chemist on 05/01/2019 16:33:53
But that's clearly nonsense.
The only "system" that we are absolutely sure is conscious - in the strongest sense- is a human being.
And in humans, the site of consciousness is the head- which isn't at the centre.

Anyway, it's got nothing to do with the topic.
Title: Re: How is the quantum double slit experiment set up?
Post by: mxplxxx on 06/01/2019 00:35:23
The only "system" that we are absolutely sure is conscious - in the strongest sense- is a human being.
And in humans, the site of consciousness is the head- which isn't at the center.
A human being is many systems. Respiratory system, vestibular system etc. etc.. No-one knows for sure what consciousness is let alone where it is located. Irrespective, I am talking about every system in the universe having awareness. We will likely have a consciousness system and in the center of this system will be a consciousness object and in the center of this object will be the current state of our consciousness. Definitely off-topic and I will continue with this in a future post.
Title: Re: How is the quantum double slit experiment set up?
Post by: mxplxxx on 10/01/2019 07:17:18
From https://www.reddit.com/r/askscience/comments/9yoxfw/what_does_it_mean_that_a_photon_has_an/ (https://www.reddit.com/r/askscience/comments/9yoxfw/what_does_it_mean_that_a_photon_has_an/):

"Sorry I don’t really understand EM waves at all and I’m probably dumb for trying to understand with a cursory knowledge of physics.

Don't worry, the EM field is extremely hard to visualise and it's very common to get confused when thinking about it. I think one of the main problems is that people imagine something like this https://i.imgur.com/7VKeLE9.png (https://i.imgur.com/7VKeLE9.png)when they think of an EM wave (or just one of E or M in this case). In fact, you should be thinking of something like this https://i.imgur.com/EJ7IjEi.png (https://i.imgur.com/EJ7IjEi.png), where the brightness of the orange represents the field strength. Note that the wave never leaves the axis -- this thing is not oscillating in some position dimension like a water wave.

The confusion generally arises because the electric and magnetic fields are vector fields, which means that in addition to the strength (shown in the second image) they also have a direction at each point. This usually leads to a picture like the first one, but that picture is just wrong. The direction doesn't mean the wave actually traces out a wave in space, it's just an abstract property of the field. In general, I wouldn't bother trying to visualise this bit; just remember that it is what leads to the polarisation. For all intents and purposes, the electric and magnetic parts of an EM wave fully overlap: they both trace out the same straight line in space."


Finally, after a very long search, something that makes an EM wave just about understandable.
Title: Re: How is the quantum double slit experiment set up?
Post by: Bored chemist on 10/01/2019 10:11:19
No-one knows for sure what consciousness is let alone where it is located.
grown ups know where it is located because we know that banging your head causes a loss of consciousness, but banging your foot doesn't.

If you don't realise that you have pretty much ruled yourself out of any sensible discussion.
I am talking about every system in the universe having awareness
Three is no evidence that such awareness exists- it's just stuff you have made up.
We will likely have a consciousness system and in the center of this system will be a consciousness object and in the center of this object will be the current state of our consciousness.
There is no reason to suppose that's "likely" and the very limited evidence we have (people's heads are not in their centre) suggests you are wrong.

Just because you believe something, doesn't make it "likely"
Finally, after a very long search, something that makes an EM wave just about understandable.

It may be understandable but, since it doesn't explain polarisation, it doesn't represent reality; it's wrong.
Title: Re: How is the quantum double slit experiment set up?
Post by: mxplxxx on 10/01/2019 13:03:01
According to this very reliable source https://www.quora.com/How-does-the-equation-E-hf-mean-that-visible-light-is-a-particle-and-a-wave-at-the-same-time (https://www.quora.com/How-does-the-equation-E-hf-mean-that-visible-light-is-a-particle-and-a-wave-at-the-same-time),

"By accepting that photons, electrons and other tiny things are simply different - neither wave nor particle, you avoid all the mystical nonsense of how does the photon know when to act as a particle and when to ac as a wave. It also avoids the double slit nonsense about how does the photon know if you are looking to see which slit it went through. "
Title: Re: How is the quantum double slit experiment set up?
Post by: Bored chemist on 10/01/2019 13:37:25
According to this very reliable source https://www.quora.com/How-does-the-equation-E-hf-mean-that-visible-light-is-a-particle-and-a-wave-at-the-same-time (https://www.quora.com/How-does-the-equation-E-hf-mean-that-visible-light-is-a-particle-and-a-wave-at-the-same-time),

"By accepting that photons, electrons and other tiny things are simply different - neither wave nor particle, you avoid all the mystical nonsense of how does the photon know when to act as a particle and when to ac as a wave. It also avoids the double slit nonsense about how does the photon know if you are looking to see which slit it went through. "
Yes, that's right, photons are not classical particles or waves. They are some odd quantum entity
And their behaviour can be calculated from the wavefunction that represents them.

It seems a pity it took you so long to realise this.

Now, if you can just ditch  "all the mystical nonsense of" centres of consciousness, you can move on.
Title: Re: How is the quantum double slit experiment set up?
Post by: mxplxxx on 10/01/2019 20:08:23
It seems a pity it took you so long to realise this.
It took me so long to realise this mainly because much of the physics community seems to be rooted in obsolete theories. Plus explanations of physics to lay-people are few and far between and those that do exist tend to be very poor quality. I am still having only the occasional eureka moment and as a result I have QED by Richard Feynman on order.
Title: Re: How is the quantum double slit experiment set up?
Post by: Bored chemist on 10/01/2019 20:58:54
much of the physics community seems to be rooted in obsolete theories.
Like what?
The post you cited is pretty much the standard view of duality.
That's a great book but... published in 1985, so it's full of "obsolete theories".
Title: Re: How is the quantum double slit experiment set up?
Post by: mxplxxx on 10/01/2019 21:11:23
Like what?
Wave/particle duality of a photon. Probability wave and dual slit experiment. Electromagnetic Wave as the wave function of the photon.As mentioned in reference. Whatever, physics really need a body that lists along with their current status the theories of physics.
QED book was published in 2014.

QED
The Strange Theory of Light and Matter (Princeton Science Library)
By Richard P. Feynman, Anthony Zee (Introduction by)
Rating
 13,932 Ratings by Goodreads
Already own it? Write a review
Format
Paperback, 192 pages
Published
United States, 1 October 2014
Title: Re: How is the quantum double slit experiment set up?
Post by: mxplxxx on 31/01/2019 09:07:46
Actually, all photons have the same energy (Planck's constant joules), but different power. Energy comes in units called quanta - and this is the basis of quantum theory. A photon always contains one quantum of energy. A lower power photon will not knock an electron from a black body but a higher power one will. See  https://www.google.com/search?rlz=1C1CHBD_en-GBAU826AU826&ei=3xccXKyMJsSS9QOi2IfYAg&q=plancks+constant+energy+of+one+cycle&oq=plancks+constant+energy+of+one+cycle&gs_l=psy-ab.3...584984.596812..597188...0.0..0.415.9758.2-30j5j1....2..0....1..gws-wiz.......0j0i71j0i67j0i131j0i10j0i22i30j0i13j0i8i13i30j0i22i10i30j33i22i29i30j33i160.Q_EF2BHDRkU (https://www.google.com/search?rlz=1C1CHBD_en-GBAU826AU826&ei=3xccXKyMJsSS9QOi2IfYAg&q=plancks+constant+energy+of+one+cycle&oq=plancks+constant+energy+of+one+cycle&gs_l=psy-ab.3...584984.596812..597188...0.0..0.415.9758.2-30j5j1....2..0....1..gws-wiz.......0j0i71j0i67j0i131j0i10j0i22i30j0i13j0i8i13i30j0i22i10i30j33i22i29i30j33i160.Q_EF2BHDRkU)
In computer science all objects in a system are types of a basic objects. It seems to me reality could follow a similar pattern where the basic object is a quantum of energy. All variations in the energy of particles could could come about via the spin of the particle (not sure about this) and its momentum (which, being relative, means particles can have any energy that is a multiple of the basic quantum of energy).
Title: Re: How is the quantum double slit experiment set up?
Post by: mxplxxx on 01/02/2019 03:37:16
So its up to me to say something new
No its not. Not your topic. Moderator, why is this off-topic garbage being allowed?
Title: Re: How is the quantum double slit experiment set up?
Post by: Bored chemist on 01/02/2019 18:20:33
In computer science all objects in a system are types of a basic objects.
And in the real universe things are not the same as they are in computer programs so....
Wave/particle duality of a photon.
It's hardly an issue for debate.
Depending on the circumstances, photons behave like particles or like waves.
That's not the same as saying they are waves, or that they are particles, or that they are some "mixture" of both.
It's clear that they are neither.
That strange behaviour is what "Duality" means.
Title: Re: How is the quantum double slit experiment set up?
Post by: mad aetherist on 01/02/2019 20:38:35
In the famous double slit experiment of quantum physics a photon fired at a wall with two slits in it appears to behave as a wave.
(1) Is this at all dependent on where the photon is fired from?
(2) For example, If the photon is fired at the part of the wall between the two slits, is it absorbed by the wall?
(3) If it is always fired at the middle of a slit, does it always pass through that slit?
(4) What is the wavefunction of the photon in the experiment?
(5) What does the wave in the experiment look like?
I would like to know your choice of which answers to the five questions were best.
Title: Re: How is the quantum double slit experiment set up?
Post by: mxplxxx on 02/02/2019 01:22:31
In the famous double slit experiment of quantum physics a photon fired at a wall with two slits in it appears to behave as a wave.
(1) Is this at all dependent on where the photon is fired from?
(2) For example, If the photon is fired at the part of the wall between the two slits, is it absorbed by the wall?
(3) If it is always fired at the middle of a slit, does it always pass through that slit?
(4) What is the wavefunction of the photon in the experiment?
(5) What does the wave in the experiment look like?
I would like to know your choice of which answers to the five questions were best.
Sorry to say, I received no answers to these questions that were satisfactory. Physics seems to be in a state of flux at present.
Title: Re: How is the quantum double slit experiment set up?
Post by: mad aetherist on 02/02/2019 02:15:37
In the famous double slit experiment of quantum physics a photon fired at a wall with two slits in it appears to behave as a wave.
(1) Is this at all dependent on where the photon is fired from?
(2) For example, If the photon is fired at the part of the wall between the two slits, is it absorbed by the wall?
(3) If it is always fired at the middle of a slit, does it always pass through that slit?
(4) What is the wavefunction of the photon in the experiment?
(5) What does the wave in the experiment look like?
I would like to know your choice of which answers to the five questions were best.
Sorry to say, I received no answers to these questions that were satisfactory. Physics seems to be in a state of flux at present.
Yes thats the impression of got when i read all of the replies. I dont know much about the single slit & double slit & triple slit experiments, with wide slits & with narrow slits, etc. And i dont know whether u need some good info to help with formal studies (in which case for sure i cant help)(i aint a scientist) or whether u are interested in science (yes i meant that to be funny in a serious way). 

Slit theory is just one more area that shows again how standard science has no good answers, nor good questions. Standard science doesnt have a clue re the nature of a photon, how big, how long, how many waves. And anyhow what is a photon wave.
Worse than that, standard science has no good ideas re the nature of em radiation.  They reckon that em radiation is made of photons. It cant get worse than that.

My theory re the aether nature of photons explains that (i) the slowing of light near mass (Shapiro Delay) & (ii) the bending of light near mass (Einstein's 1.75 arcsec) & (iii) the bending of light in mass (Snell's refraction) & (iv) the bending of light in slits (diffraction) & (v) em radiation, are all due to one cause.  But that wont help exams.
Title: Re: How is the quantum double slit experiment set up?
Post by: mxplxxx on 02/02/2019 10:04:32
My theory re the aether nature of photons explains that (i) the slowing of light near mass (Shapiro Delay) & (ii) the bending of light near mass (Einstein's 1.75 arcsec) & (iii) the bending of light in mass (Snell's refraction) & (iv) the bending of light in slits (diffraction) & (v) em radiation, are all due to one cause.  But that wont help exams.
Where can I find your theory? I would consider ditching the use of the word, aether, which has slightly crackers connotations. I am a software developer who thinks the universe is an abstraction hierarchy (a sort of finite state machine) and am developing a software framework, 3dAbstractions,  that implements this hierarchy. Photons in the framework are events that transfer state between objects/particles.
Title: Re: How is the quantum double slit experiment set up?
Post by: Bored chemist on 02/02/2019 12:20:57
Sorry to say, I received no answers to these questions that were satisfactory.
They were "unsatisfactory" because you refused to understand them.


Physics seems to be in a state of flux at present.
It seems that way to you, but not to those who understand it..

dont know much about the single slit & double slit & triple slit experiments, with wide slits & with narrow slits, etc.
And yet, you thought you were qualified to comment on them.
What does that tell us about you?
I would consider ditching the use of the word, aether, which has slightly crackers connotations.
The word is entirely appropriate.

I am a software developer who thinks the universe is an abstraction hierarchy (a sort of finite state machine) and am developing a software framework, 3dAbstractions,  that implements this hierarchy.
There is no evidence for the underlying belief there,
Title: Re: How is the quantum double slit experiment set up?
Post by: mad aetherist on 02/02/2019 21:59:29
dont know much about the single slit & double slit & triple slit experiments, with wide slits & with narrow slits, etc.
And yet, you thought you were qualified to comment on them. What does that tell us about you?
Re reality & models & mathland, i am thinking that the mathland wave-models re fringes at slits might be ok (if they give good answers), & the mathland quantum models might be ok (if they give good answers), but i was merely trying to introduce some reality.  What we have is a physical photon being diffracted by the slit. My idea is photons have a main central helical body (that propagates at c throo the aether) & lots of photaenos (tornadic swirls of the aether) that emanate from the body (probly at 5c). Thusly a photon is very large.  If photaenos are slowed by the presence of mass etc then that slows the whole photon, mainly on that side, the photon veers, it diffracts. So here u have a physics, a mechanical explanation, a rare animal in physics nowadays.  I doubt that this simple reality will help much, ie it wont lead to better equations, but it is a start.
And it wont help mxplxxx to construct his computer model. The beauty of a simple model is that it allows u to make a computer program using lots of iterations of small equations, rather than messing around with integration & a big equation.  But i doubt that my photon-photaeno reality will yield any kind of simple model-program. But u never know.
Title: Re: How is the quantum double slit experiment set up?
Post by: mad aetherist on 05/02/2019 23:53:37
My theory re the aether nature of photons explains that (i) the slowing of light near mass (Shapiro Delay) & (ii) the bending of light near mass (Einstein's 1.75 arcsec) & (iii) the bending of light in mass (Snell's refraction) & (iv) the bending of light in slits (diffraction) & (v) em radiation, are all due to one cause.  But that wont help exams.
Where can I find your theory? I would consider ditching the use of the word, aether, which has slightly crackers connotations. I am a software developer who thinks the universe is an abstraction hierarchy (a sort of finite state machine) and am developing a software framework, 3dAbstractions,  that implements this hierarchy. Photons in the framework are events that transfer state between objects/particles.
If u search New Theories for photaenos u will find most of my thinking.  Which is still developing, & i will add to it during the coming weeks.
But how will your software program work?  Do u simply crunch the math, or  do u try to set up a simple mechanical model & then apply & iterate & sum etc a very simple equation umpteen times. 
I have tried iteration using Excel for the motion & impact & rebound of billiards balls & cushion-rails.

Will your program simply give a number, or will it give a curve or trajectory of some kind of movement or quantity?
Title: Re: How is the quantum double slit experiment set up?
Post by: Bored chemist on 06/02/2019 19:57:44
dont know much about the single slit & double slit & triple slit experiments, with wide slits & with narrow slits, etc.
And yet, you thought you were qualified to comment on them. What does that tell us about you?
Re reality & models & mathland, i am thinking that the mathland wave-models re fringes at slits might be ok (if they give good answers), & the mathland quantum models might be ok (if they give good answers), but i was merely trying to introduce some reality.  What we have is a physical photon being diffracted by the slit. My idea is photons have a main central helical body (that propagates at c throo the aether) & lots of photaenos (tornadic swirls of the aether) that emanate from the body (probly at 5c). Thusly a photon is very large.  If photaenos are slowed by the presence of mass etc then that slows the whole photon, mainly on that side, the photon veers, it diffracts. So here u have a physics, a mechanical explanation, a rare animal in physics nowadays.  I doubt that this simple reality will help much, ie it wont lead to better equations, but it is a start.
And it wont help mxplxxx to construct his computer model. The beauty of a simple model is that it allows u to make a computer program using lots of iterations of small equations, rather than messing around with integration & a big equation.  But i doubt that my photon-photaeno reality will yield any kind of simple model-program. But u never know.
So, you were unable to actually answer the question...
Title: Re: How is the quantum double slit experiment set up?
Post by: mad aetherist on 06/02/2019 23:57:10
dont know much about the single slit & double slit & triple slit experiments, with wide slits & with narrow slits, etc.
And yet, you thought you were qualified to comment on them. What does that tell us about you?
Re reality & models & mathland, i am thinking that the mathland wave-models re fringes at slits might be ok (if they give good answers), & the mathland quantum models might be ok (if they give good answers), but i was merely trying to introduce some reality.  What we have is a physical photon being diffracted by the slit. My idea is photons have a main central helical body (that propagates at c throo the aether) & lots of photaenos (tornadic swirls of the aether) that emanate from the body (probly at 5c). Thusly a photon is very large.  If photaenos are slowed by the presence of mass etc then that slows the whole photon, mainly on that side, the photon veers, it diffracts. So here u have a physics, a mechanical explanation, a rare animal in physics nowadays.  I doubt that this simple reality will help much, ie it wont lead to better equations, but it is a start.
And it wont help mxplxxx to construct his computer model. The beauty of a simple model is that it allows u to make a computer program using lots of iterations of small equations, rather than messing around with integration & a big equation.  But i doubt that my photon-photaeno reality will yield any kind of simple model-program. But u never know.
So, you were unable to actually answer the question...
We could argue about the quality of desirable qualifications, or we could argue about the quality of comments. Getting on your hi-horse & shirking comments -- what does that tell us about u? I picture a portrait of a big dictator with a funny hat mounted on a child's rocking horse.
Title: Re: How is the quantum double slit experiment set up?
Post by: mad aetherist on 07/02/2019 01:13:01
Miles Mathis – The Double Slit Experiment. http://milesmathis.com/double.html
Miles mentions physical mechanical & math models.
First posted August 25, 2008.
In this paper I will show the simple mechanical solution to the famous double slit (or two slit) experiment. Feynman called it the most important experiment for understanding quantum motion, and he may have been right. The most difficult problems are always the most important, and this one has remained unsolved up to this minute. Thomas Young first performed it in 1801, which means it has been a mystery for over 200 years. Even Feynman failed to solve it. He offered a mathematical solution only, but was not able to provide a physical solution.
I will solve the two biggest problems here: the problem of the single photon and the problem of the detector. In the first problem, we let photons go through the experiment one at a time. Using the photon-as-particle theory that Einstein proved and Feynman confirmed, we expect no wave interference, since the photon must go through one slit or the other. But we see interference. The single photon seems to be interfering with itself in some strange way. Up till now, there have been several proposed solutions. Wikipedia lists the most important. The first they mention is that the “wave front” of the photon goes through both slits and interferes with itself. Since the wave front is still not defined mechanically, this solution is not very compelling. The wave front is and always has been defined using Huygen’s visualization. The wave is seen as a semi-circular forward transmission from every point on a line of moving photons. With a single photon, this would be a semi-circle in front of the photon. But we are never told how far this semi-circle extends, what it is composed of, or how it acts upon the field. So we will let this explanation pass as wholly unsatisfactory.
Another solution is to define the photon as a probability. A discrete particle cannot go through both slits at once, but a particle as probability can (as long as we define probabilities in certain ways).
The third and currently accepted explanation is an extension of this second one. Feynman proposed that the photon-as-probability traveled every possible path, and therefore through both slits. Each path is given an equation, and we “sum over” all these equations. If we sum over in the correct way, we achieve interference.
Now, admittedly this is a clever mathematical solution. Feynman was a master of clever mathematical solutions, and this is one of his best. Mathematically it works. But it is not a physical or mechanical solution. It is a mathematical solution. Feynman was not so much a physicist as he was a mathematician that had invaded the physics department (the same could be said of most modern physicists). In his own way, Feynman admitted this. He did not admit to being an invader, but he admitted that his solution was only mathematical. He knew as well as anyone that it wasn’t physical, by the old definition of physics. He got around this by claiming that new physics was and must be mathematical only, since there was no possible mechanical solution.
He was wrong, as I will prove very quickly. His math works precisely because there is a physical reality underlying his probabilities. Probabilities are not the causa sui, the cause of themselves. It is illogical—even as a piece of mathematics—to propose that probabilities are spontaneously generated, or that they are primary generators. No, they must be generated by a real field. Even in pure mathematics, probabilities are always secondary numbers, produced by an underlying field of numbers. A field of probabilities cannot be a foundational field. They therefore cannot take the place of a physical field.